Moving Charges and Magnetism 2

You might also like

Download as pdf or txt
Download as pdf or txt
You are on page 1of 43

CLASS – 12

100
WORKSHEET- MOVING CHARGES AND MAGNETISM

A. BIOT-SAVART LAW

(1 Mark Questions)

1. A circular coil of wire consisting of 100 turns each of radius 8.0 cm carries a current of
0.40A, what is the magnitude of magnetic field B at the centre of the coil?
Sol. Given N = 100, r = 8cm = 0.08m, I = 0.40A
μ0 NI 4π×10−7 ×100×0.40
∴B= = =  × 10-4 = 3.1 × 10-4 T.
2r 2×0.08

2. A long straight wire carries a current of 35A. What is the magnitude of the field B at a
point 20cm from the wire?
Sol. Here I = 35A, r = 20c = 0.20m,  = 4 ×10-7 mA-1
μ I
0 4π×10−7×35
B = 2πr = = 3.5×10-5T.
2π×0.20

3. A long straight wire in the horizontal plane carries a current of 50A in the north to south
direction. Give the magnitude and direction of B at a point 2.5m east of the wire.
Sol. Here I = 50A, r = 2.5m
μ I
0 4π×10−7 ×50
B = 2πr = = 4 × 10-6T.
2π×2.5
Applying right hand thumb rule, we find that the magnetic field will act in the vertically
upward direction at the point 2.5m east of the wire.

4. A horizontal overhead power line carries a current of 90A in an east to west direction.
What is the magnitude and direction of magnetic field due to the current 1.5m below the
line?
Sol. Here I =- 90A, r = 1.5m,  = 4 ×10-7 TmA-1
μ I
0 4π×10−7×90
B = 2πr = T = 1.2 × 10-5 T.
2π×2.5
Applying right hand thumb rule, we find that the direction of the field B will be towards
south at a point below the power line.

5. State Biot-Savart’s law.


Sol. ⃗⃗⃗ at the
According to Biot-Savart law, the magnetic field due to a current element I.dl
⃗⃗⃗
⃗ = μ0I . dl×r
observation point whose position vector is r is given by d.B

where 0 is the
4π r3
permeability of free space.

Er. Ujwal Kumar (Physics Mentor for NEET/ JEE-Mains, Adv/ KVPY/OLYMPIAD/CBSE)
6. What is SI unit of 0?
Sol. SI unit of permeability (0) = TmA-1 or WbA-1 m-1. 101

7. What is the effect of increasing number of turns n magnetic field produced to a circular
coil?
Sol. For a circular coil, B ∝ N. As the number of turns increases, the magnetic field increases.

8. What is force experienced by a stationary charge in a magnetic field?


Sol. For a stationary charge, v = 0. Therefore F = qvB sin = q(0) B sin = 0.

9. What is the work done by magnetic field on a moving charge?


Sol. Zero, because a magnetic force acts perpendicular to the direction of velocity or the
direction of motion fo the charged particle.

10. Biot-Savart Law indicates that the moving electrons (velocity v) produce a magnetic field
B such that
(a) B ┴ v (b) B||v (c) it obeys inverse cube law
(d) It is along the line joining the electron and point of observation
Ans. (a)
Id/sinθ I×d𝑙
. By Biot-Savart law, dB = r2 = ( r )
In Biot Savart law, magnetic field B||idl × r and idl due to flow of electron is in opposite
direction of v and by direction of cross product of two vectors
B⊥V
So, the magnetic field is ⊥ to the direction of flow of charge

(2 Marks Questions)

11. Two very small identical circular loops,(1) and (2), carrying equal currents I are placed
vertically (with respect to the plane of the paper) with their geometrical axes
perpendicular to each other as shown in the magnitude and direction of the net magnetic
field produced at the point O.

Sol. The magnetic field at an axial point due to a circular loop is given by
μ 2πIa2
B = 4π0 (a2 +r2 )3/2 where I = current through the loop, a = radius of the loop, r = distance
of O from the centre of the loop.

Er. Ujwal Kumar (Physics Mentor for NEET/ JEE-Mains, Adv/ KVPY/OLYMPIAD/CBSE)
Since I, a and r = x are the same for both the loops, the magnitude of B woill be same and
μ 2πIa2 102
is given by B1 – B2 = 4π0 (a2 +r2)3/2
The direction of magnetic field due to loop (1) will be away from O and that of loop (2)
will be towards O as shown. The direction of the net magnetic field will bas as shown in
figure.

The magnitude of the net magnetic field is given by B net = √B12 + B22
μ 2√2πIa2
Bnet = 4π0 (a2 +x2 )3/2

(3 Marks Questions)
12. (a) State biot-savart law and express this law in the vector form.
(b) Two identical circular coils, P and Q each of radius R, carrying current 1 A and
3 A respectively; are placed concentrically and perpendicular to each other lying in the
XY and YZ planes. Find the magnitude and direction of the net magnetic field at the
Centre of the coils.
Sol. (a) A current carrying wire produces a magnetic field around it. Biot-Savart law states
⃗ due to current I carrying element
that magnitude of intensity of small magnetic field d𝐵
d𝑙 at any point P at distance r fro it is given by
⃗ | = μ0 Idlsinθ
| d𝐵 2 where  is the angle between 𝑟 and d𝑙 and 0 = 4 × 10-7 TmA-1 is called
4πr
permittivity of free space.
⃗ = μ0 I dl×r
In vectorial form, d𝐵 4π r3
⃗ is perpendicular to the plane containing 𝑟 and d𝑙.
So, the direction of d𝐵
SI unit of magnetic field strength is tesla denoted by “T” and cgs unit is gauss determined
by ‘G’, where 1T = 104G.
(b) Field due to current in coil P is
⃗ 1 = μ0 I1 . k̂
𝐵 2R
⃗ 2 = μ0I2 .î
Current in coil Q is 𝐵 2R
∴ Net field 𝐵 ⃗ =𝐵
⃗1+𝐵 ⃗2
μ0 I 1 μ0 I 2
∴𝐵⃗ = ( ) k̂ + ( ) î
2R 2R

Er. Ujwal Kumar (Physics Mentor for NEET/ JEE-Mains, Adv/ KVPY/OLYMPIAD/CBSE)
2 2
μ √3μ0
= (2R0 ) + ( ) ( ∵ I1= 1A, I2 = √3A) 103
2R
2 2
⃗ = √( μ0 ) + (√3μ0 )
∴𝐵
2R 2R
μ0 μ0
= 2R √1 + 3 = 2R × 2
μ
⃗ = 0
∴𝐵 2R
The resultant magnetic field is directed to XZ plane.

13. Two identical circular loops P and Q each of radius r and carrying equal currents I are
kept in the parallel planes having a common axis passing through O. the direction of
current in P is clockwise and in Q is anti- clockwise as seen from O which is equidistant
from the loops P and Q . find the magnitude of the net magnetic field at O.

Sol. The magnetic field at a point on the axis of a circular point on the axis from the centre of
the loop:

0 μ Ir2 μ Ir2
0 μ I
0
A magnetic field at O due to loop P is BP = 2(r2 +r 2 )3/2 = 2(2r2)3/2 = 2(2)3/2 r

By the right hand thumb’s rule, the dimension of magnetic field will be towards left.
0 μ Ir2 0μ Ir2 0μ I
Magnetic field at O due to loop Q is BQ = 2(r2+r 2 )3/2 = 2(2r2)3/2 = 2(2)3/2r

By the right hand thumb’s rule, the direction of the magnetic field will be toward left.
Since BP and BQ are equal in magnitude and in same direction. Therefore, net magnetic
μ0 I
field at O is ⃗B = ⃗BP + ⃗BQ = 3/2 2 r

14. A straight wire carrying a current of 12A is bent into a semicircular arc of radius 2.0 as
shown in the figure (a). What is the direction and magnitude of B at the centre of the arc?
Would your answer change if the wore were bent into a semicircular arc of same radius
but in the opposite way as shown in the figure (b)?

Er. Ujwal Kumar (Physics Mentor for NEET/ JEE-Mains, Adv/ KVPY/OLYMPIAD/CBSE)
104

μ0 I
Sol. (a) magnetic field at the center of the arc is B = . Here I = 12A, r = 3.0cm = 0.02m, 0
4r
= 4 × 10-7 TmA-1.
4π×10−7×12
Therefore, B = = 1.9×10-4T
4×0.02
According to right hand rule, the direction of the field is normally into the plane of
paper.
(b) The magnetic field will be of same magnitude, B = 1.9 × 10 -4T. The direction of the
field is normally out of the plane of paper.

15. Figure shows a current loop having two circular segments and joined by two radial lines.
Find the magnetic field at the centre O.

Sol. Since the point O lies on lines SP and QR, so, the magnetic field at O due to these
straight portions is zero.
μ I
The magnetic field at O due to the circular segment PQ is B1 = 4π0 a2 l, Here l is the length
of arc PQQ = a.
μ Iα
∴ B1 = 4π0 , directed normally upward.
a
μ Iα
Similarly the magnetic field at O due to circular segments SR is, B2 = 4π0 directed
b
normally downward.
μ0 Iα 1 1
The resultant field at O is, B = B1 – B2 = [ − ]
4π a b
μ0 Iα(b−a)
Or B = 4πab

16. A circular segment of radius10cm subtends an angle of 60° at its centre. A current of 9A
is flowing through it. Find the magnitude and direction of the magnetic field produced at
the centre. [Ans. 9.42×10-6 T]

Er. Ujwal Kumar (Physics Mentor for NEET/ JEE-Mains, Adv/ KVPY/OLYMPIAD/CBSE)
105

Sol. Here  = 60° = /3 rad


As  (rad) = 1/r, ∴ /3 = l/r or l = r/3
According to Biot-Savart law, magnetic field at centre O is
μ Il μ I πr μ π I 10−7×3.14×9
B = 4π0 . r2 = 4π0 . r2 . = 4π0 . 3 . r = = 9.42×10-6T
3 3×0.10

17. In the figure, the curved portion is a semicircle and the straight wires are long. Find the
magnetic field at the point O.

Sol. Magnetic field at point O due to any current element is perpendicular to and points out of
the plane of paper.
1 μ0 I 0 μ I
Magnetic field at O due to upper straight wire is B 1 = 2 × d = 2πd
2π( )
2
μ0 I
Similarly field at O due to lower straight wire is B2 =
2πd
1 μ0 I μ0 I
Field at O due to the semicircle of radius d/2 is B3 = 2 × d =
2( ) 2d
2
μ I 2
Resultant field at O, B = B1 + B2+ B3 = 2d0 [1 + π]

18. A metallic wire is bent into the shape shown in the figure and carries a current I. If O is
the common centre of all the three circular arcs of radii r, 2r and 3r, find the magnetic
field at the point O.

Er. Ujwal Kumar (Physics Mentor for NEET/ JEE-Mains, Adv/ KVPY/OLYMPIAD/CBSE)
106

Sol. Magnetic field at O due to straight parts of the wire will be zero. Magnetic fields at O
due to the three circular arcs of radii r, 2r and 3r are
μo I θ
B1 = . , acting normally inward
4π r
μo I θ
B2 = . , acting normally outward
4π 2r
μo I θ
B3 = . , acting normally inward
4π 3r
Thus the total magnetic field at the centre O is
μo I θ θ θ 5μ I
B = B1 – B2 + B3 = o
( r − 2r + 3r) = 24πr θ, acting normally inward.

19. Two insulating infinitely long conductors carrying currents I 1 and I2 lie mutually
perpendicular to each other in the same plane as shown in figure. Find the locus of the
point at which the net magnetic field is zero.

Sol. According to right hand thumb rule, the magnetic fields due to the two conductors can
vanish only in regions I and III.

Er. Ujwal Kumar (Physics Mentor for NEET/ JEE-Mains, Adv/ KVPY/OLYMPIAD/CBSE)
Let the magnetic field be zero at point P(x, y)
Magnetic field at P due to current I1, B1 =
μ0 I 1
(directed inwards) 107
2πx
μ0 I 2
Magnetic field at P due to current I2, B2= 2πy (directed outwards)
As the net magnetic field at P is zero, so B1 = B2
μ0 I 1 μ0 I 2 I
= or y = I1 . x.
2πx 2πy 2

20. As shown in the figure, a cell is connected across two points A and B of a uniform
circular conductor. Prove that the magnetic field at its centre O will be zero.

Sol. Let the lengths of the two circular segments ACB and ADB be l1 and l2, and  be the
resistance per unit length. Then
Resistance of segment ACB , R1 = l1
Resistance of segment ADB, R2 = l2
Suppose I1 and I2 are the currents in segments ACB and ADB respectively. As the two
segments are connected in parallel, so the potential differences across them must be
equal.
∴ I1R1 = I2R2
Or O1l1 = I2l2 or I1l1 = I2l2
μ0 Il
Magnetic field at the centre of an arc of length l carrying current I is, B = 4πr2
Therefore, the magnetic fields at the centre O due to the current I1 and I2 are
μ0 I1l1 μ0 I2l2
B1 = and B2 = where r is the radius of the circular conductor. As I 1l1 = I2l2,
4πr2 4πr2
so, B1 = B2.
As the currents I1 and 2 are oppositely directed, their magnetic fields B1 and B2 will be
opposite to each other. Hence the resultant field at the center O is zero.

(5 Marks Questions)

21. Two identical coils P and Q each of radius R are lying in perpendicular planes such that
they have a common center. Find the magnitude and direction of the magnitude and

Er. Ujwal Kumar (Physics Mentor for NEET/ JEE-Mains, Adv/ KVPY/OLYMPIAD/CBSE)
direction of the magnetic field at the common center of the two coils, if they carry
currents equal to I 3 I respectively. 108

μ0 I
Sol. Magnetic field at the center of the coils due to the coil P, having current I is B P = 2R
μ0 √3I
And magnetic field due to coil Q having current √3I is BQ = 2R
Since both coils are inclined to each other at an angle of 90°, the magnitude fo their
μ0 I μ0 I
resultant magnetic field at the common centre will be B = √BP2 + BQ2 = √1 + 3 =
2R R

The directions of BP and BQ are as indicated in the figure. The direction of their resultant
B 1
field is at an angle  given by  = tan-1 ( P ) = tan−1 ( ) = 30°
BQ √3
Hence the direction of the magnetic field will be at an angle 30° to the plane of loop P.

22. A circular loop of radius R carries a current I. obtain an expression for the magnetic field
at a point on its axis at a distance x from its center.
Sol. Magnetic field on the axis of circular coil

Small magnetic field due to a current element of circular coil of radius r at point P at
μ Idlsin90° μ Idl
distance x from its centre is dB = 4π0 = 4π0 (r2+x2 )
S2

Er. Ujwal Kumar (Physics Mentor for NEET/ JEE-Mains, Adv/ KVPY/OLYMPIAD/CBSE)
Components dBcos due to current element at point P is canceled by equal and opposite
component dBcos of another diametrically opposite current element, where the sine 109
components dBsin add up to give net magnetic field along the axis. So, net magnetic
2πr μ0 Idl r
field at point P due to entire loop is B = ∮ dBsin∅ = ∫0 .
4π (r2+x2 ) (r2+x2 )3/2
μ Ir
0 2πr
B = 4π(r2 +x2 )3/2 ∫0 dl
0μ Ir
Or B = 4π(r2+x2 )3/2 . 2πr

μ0Ir2
Or B = directed along the axis (a) towards the coil if current in it is in
2(r2+x2 )3/2
clockwise direction. (b) away from the coil if current in it is in anticlockwise direction.

23. Two concentric circular coils X and Y of radii 16cm and 10cm respectively lie in the
same vertical plane containing the north-south direction. Coil X has 20 turns and carries a
current of 16A; coil Y has 25 terms and carries a current of 18A. The sense of current in
X is anticlockwise, and in Y clockwise, for an observer looking at the coils facing west.
Give the magnitude and direction of the net magnetic field due to the coils at their centre.
Sol.

For coil X, rx = 16cm = 0.16m, Nx = 20, Ix = 16A


μ0 IxNx 4π×10−7 16×20
∴ Magnetic field at the centre of coil X is B x = = × T = 4 × 10-4T
2rx 2 0.16
As the current in the coil X is anticlockwise, the field is directed towards east.
For coil Y: ry = 10cm= 0.10m, Ny – 25, I = 18A
μ0 IyNy 4π×10−7 18×25
∴ Magnetic field at the centre of coil Y is B y = = × T = 9 × 10-4T
2ry 2 0.10
As the current in the coil Y is clockwise the field By is directed towards west. Since
By>Bx, therefore the net field is directed towards west and its magnitude is
B = By – Bx = 5 × 10-4T = 1.6 × 10-3 T.

B. AMPERE’S CIRCUITAL LAW

(1 Mark Question)

1. Show the magnetic lines of force around a straight current carrying conductor.

Er. Ujwal Kumar (Physics Mentor for NEET/ JEE-Mains, Adv/ KVPY/OLYMPIAD/CBSE)
______________________________________________________________________________
______________________________________________________________________________ 110

2. How does current carrying coil behave like a bar magnet?


______________________________________________________________________________
______________________________________________________________________________

(2 Marks Questions)

3. A closely wound solenoid 80cm long has 5 layers of windings of 400 turns each. The
diameter of the solenoid is 1.8cm. If the current carried is 8.0A, estimate the magnitude
of B inside the solenoid near its centre.
Sol. Number of turns per unit length of the solenoid is
Number of turns per layer×Number of layers 400×5
n= = = 2500m-1
Length of solenoid 0.80
Magnetic field inside the solenoid is, B = 0nI = 4 × 10-7 × 2500 × 8 = 8 × 10-3T
= 2.5 × 10-2T

(3 Marks Questions)

4. Explain how biot-savart law enables one to express the Ampere’s circuital law in the
integral form, viz.
B. d l= 0I
∮ ⃗⃗⃗
where I is the total current passing through the surface.

Sol. Ampere’s circuital law states that line integral of magnetic field over a closed loop or
circuit is 0 times the total; current I threading through the loop i.e., ∮ ⃗⃗⃗
B. d l= 0I.

⃗⃗⃗ d l = Bdl cos0°


Proof: For the small element, B.
∫B⃗ . dl = ∫ Bdl = ∫ μ02I dl
4πr
μ0 2I μ0 2I
= ∫ 4π . r ∫ dl = 4π . × 2πr
r
= ∫ ⃗B. dl = 0I

Er. Ujwal Kumar (Physics Mentor for NEET/ JEE-Mains, Adv/ KVPY/OLYMPIAD/CBSE)
5. A toroid has a core (non-ferromagnetic) of inner radius 25cm and outer radius 26cm
around which 3500 turns of a wire are wound. If the current is in the wire is 11A, what is 111
the magnetic field (a) outside the toroid (b) inside the core of the toroid (c) in the empty
space surrounded by the toroid?
Sol. Here I = 11A, total number of turns = 3500
25+26
Mean radius of toroid, r = = 25.5cm = 25.5 × 10-2m
2
Length of (circumference) of the toroid = 2r = 2 ×25.5×10-2 = 51.0×10-2m
3500
∴ Number of turns per unit length, n =
51.0×10−2 π
(a) The field outside the toroid is zero
3500
(b) The field inside the core of the toroid, B = 0 nI = 4 × 10-7 × 51.0×10−2 π × 11
= 3.02 × 10-2T
(c) The field in the empty space surrounded by the toroid is zero.

6. State Ampere’s circuital law and prove this law for a circular path around a long current
carrying conductor.

______________________________________________________________________________
______________________________________________________________________________
______________________________________________________________________________
______________________________________________________________________________
______________________________________________________________________________
______________________________________________________________________________

7. Using Ampere’s circuital law, derive an expression for the magnetic field along the axis
of a current carrying toroidal solenoid of N number of turns having radius r.

______________________________________________________________________________
______________________________________________________________________________
______________________________________________________________________________
______________________________________________________________________________
______________________________________________________________________________
______________________________________________________________________________

8. Using Ampere’s circuital theorem, calculate magnetic field due to an infinitely long wire
carrying current I.

______________________________________________________________________________
______________________________________________________________________________
______________________________________________________________________________
______________________________________________________________________________

Er. Ujwal Kumar (Physics Mentor for NEET/ JEE-Mains, Adv/ KVPY/OLYMPIAD/CBSE)
______________________________________________________________________________
______________________________________________________________________________ 112
(5 Marks Questions)

9. Figure shows a long straight wire of a circular cross –section of radius ‘a’ carrying
steady current I. the current I is uniformly distributed across the cross-section. Derive the
magnetic field in the region r  a and  a .

Sol. (i) Let us consider a circular loop L1 of radius (r1 <a) inside the current carrying wire.
I Ir21
The current enclosed buy this loop L1 is I1 = πa2 × πr12 = a2
μ0 I1 r21
⃗ . dl=μ0 I1 =
So, by Ampere’s curcuital law, ∮L B …(i)
1 a2
⃗ . dl = ∮ B
Also, ∮L B ⃗ . dl cos0° = ∮ B⃗ . dl = B.2r1 …(ii)
1 L 1 L 1
So, by equations (i) and (ii)
μ0 I1 r21 μ0Ir1
B.2r1 = or Bin =
a2 2πa2
(ii) Let us consider a solid metallic wire of cross section ‘a’ carrying current I. Let us
consider a circular loop L of radius r outside the wire, representing a magnetic field line.
So, at any point on it magnetic field B is along the tangent to field line at that point.
∮ B ⃗⃗⃗ = ∮ Bdlcos0° = B ∮ dl = B.2r …(i)
⃗ . dl
L L L
⃗⃗⃗ = μ0 I …(ii)
⃗ . dl
But, by Ampere’s circuital law, ∮L B
By, equations (i) and (ii) we get
μ I μ I
B.2r = 0I or Bout = 2πr
0 0
whereas on the surface of current carrying wire B surface = 2πa

10. Two long coaxial insulated solenoid, S1 and S2equal length are wound one over the other
as shown in the figure. A steady current ‘I’ flow through the inner solenoid s 1to the other
end B, which is connected to the outer solenoid S 2through which the same current ‘I’
flows in the opposite direction so as to come out at end A. if n1 and n2are the number of
turns per unit length, find the magnitude and directions of the net magnetic field at the
point (i)inside on the axis and (ii)outside the combined system .

Er. Ujwal Kumar (Physics Mentor for NEET/ JEE-Mains, Adv/ KVPY/OLYMPIAD/CBSE)
113

Sol (i) Magnetic field due to a current carrying solenoid, B = 0nI where n = number fo turns
per unit length, I = current flowing in the solenoid
Bin = B2 – B1
⇒ Bin = 0n2I – 0n1I
⇒ Bin = 0(n2 – n1).
(ii) Magnetic field at point outside the combined system is zero.

C. LORENTZ FORCE AND MOTION OF A CHARGED PARTICLE INSIDE THE


MAGENTIC FIELD

(1 Mark Questions)

1. Write the expression, in a vector form, for the Lorentz magnetic force F due to a charge
moving with velocity 𝑣 in a magnetic field 𝐵⃗ . what is the direction of the magnetic force?
Sol. ⃗ is
The magnetic force experienced by the charge of moving velocity v in magnetic field 𝐵
⃗)
given by Lorentz, 𝐹 = q(𝑣 × 𝐵
⃗ . The
The direction of the Lorentz force is perpendicular to the plane containing 𝑣 and 𝐵
direction is given by right hand screw rule.

2. Define one tesla using the expression for the magnetic force acting on a particle of charge
‘q’ moving with velocity V in a magnetic field B.
Sol. One tesla is defined as the magnitude of magnetic field which produces a force of 1
newton when a charge of 1 coulomb moves perpendicularly in the region of the magnetic
field at a velocity of 1 m/s.
F 1N
F = qvB ⇒ B = qV or 1T = 1m .
(1C)( )
s

Er. Ujwal Kumar (Physics Mentor for NEET/ JEE-Mains, Adv/ KVPY/OLYMPIAD/CBSE)
3. A charge particle after being accelerated through a potential difference ‘V’ enters in a
uniform magnetic field and moves in a circle of radius r. if V is doubled, the radius of the 114
circle will become.
r
(a) 2r (b) 2r (c) 4r (d)
2
Sol. (b)
mv2 mv
As qvB = ⇒r= …(i)
r qB

2qV
Now qV = ½ mv2 ⇒ v = √ …(ii)
m
Substituting (ii) in (i)
m 2qV 2mV
r = qB √ = √ qB2 …(iii)
m

2m(2V) 2mV
Now, V’ = 2V, ∴ r’ = √ = √2√ qB2 = √2 r
qB2

4. A long straight wire carries a steady current I along the positive y-axis in a coordinate
system. A particle of charge +Q is moving with a velocity v along the x-axis. In which
direction will the particle experience a force?
Sol. From relation 𝐹 = qVB[𝑖̂ × (-𝑘̂)] = +qvB(𝑗̂). Magnetic force 𝐹 will be along +y axis.

5. A narrow beam of protons and deuterons, each having the same momentum, enters a
region of momentum. What would be the ratio of the circular paths describe by them?
Sol. Charge on deuteron (qd) = charge on proton (qp)
p mv2
Radius of circular path (r) = Bq (∵ qvB = )
r
1
r ∝ q [for constant momentum (p)]
r q
So, rp = qd = 1
d p

Hence rp: rd = 1: 1.

6. Write the condition under which an electron will move undeflected in the presence of
crossed electric and magnetic fields.

Er. Ujwal Kumar (Physics Mentor for NEET/ JEE-Mains, Adv/ KVPY/OLYMPIAD/CBSE)
Sol. The charged particle goes undeflected in the presence of crossed electric and magnetic
field only when both these fields are perpendicular to velocity of charged particle. In that 115
case, qE = qvB.

7. A uniform magnetic field B is set up along the positive x-axis. A particle of charge ‘q’
and mass ‘m’ moving with a velocity v enter the field at the origin in x-y plane such that
it has velocity components both along and a perpendicular to the magnetic field B. trace,
giving reason, the trajectory followed by the particle. Find out of expression for the
distance moved by the particle along the magnetic field in one rotation.
Sol. ⃗ ) ∴ |𝐹| = qvB sin 
Magnetic force an a charged particle 𝐹 = q(𝑣 × 𝐵

mvsinθ
The radius of circular path, r = qB
2πm
Time period, T = Bq
Horizontal distance moved by the particle in one rotation,
2πm
Pitch = v cos × T = cos 
Bq
Path of the charged particle will be helical.

8. Answer the following questions: (1 mark each)


(a) A magnetic field that varies in magnitude from point ort point but has a constant
direction (east to west) is set up in a chamber. A charged particle enters the chamber and
travels undeflected along a straight path with constant speed. What can you say about the
initial velocity of the particle?
Sol. The force on a charged particle moving in a magnetic field is given by F = qvBsin. The
force on a charged particle will be zero or the particle will remain undeflected if sin = 0
or  = 0°, 180°
(b) A charged particle enters an environment of a strong and non-uniform magnetic field
varying from point to point both in magnitude and direction and comes out of it following
a completed trajectory. Would its final speed equal the initial speed if it suffered no
collisions with the environment?
Sol. Yes, a magnetic field exerts force on a charged particle in a direction perpendicular to its
direction of motion and hence does no work on it. So the charged particle will have its
final speed equal to its initial speed.
(c) An electron travelling west to east enters a chamber having a uniform electrostatic
field in a north to south direction. Specify the direction in which a uniform magnetic field
should be set up to prevent the electron from deflecting from its straight line path.

Er. Ujwal Kumar (Physics Mentor for NEET/ JEE-Mains, Adv/ KVPY/OLYMPIAD/CBSE)
Sol. The electron travelling west to east experiences a force towards north due to the
electrostatic field. It will remain undeflected if it experiences an equal force towards 116
south due the magnetic field. According to Fleming’s left hand rule, the magnetic field
must act in the vertically downward direction.

9. Two charged particles traverse identical helical paths in a completely opposite sense in a
uniform magnetic field B = B0k̂
(a) They have equal z-components of moment a (b) They must have equal charges
(c) They necessarily represent a particle antiparticle pair.
(d) The charge to mass ratio satisfy: (e/m)1 + (e/m)2 = 0
Ans. (d)
As we know that the uniqueness of helical path is determined by its pitch
2πmvcosθ
P(Pitch) = Bq where  is angle of velocity of charge particle with x axis. For the
q 2πvcosθ
given pitch d correspond to charge particle, we have = = constant
m BP
If motion is not helical, ( = 0)
As charged particles traverse identical helical paths in a completely opposite direction in
a same magnetic field B. LHS for two particles should be same and of opposite sign
e e
Therefore,       0
 m 1  m 2

10. Show that a force that does not work must be a velocity dependent force.
Sol. dW = F.dI = 0
⇒ F.vdt = 0
⇒ F.v = 0
F must be velocity dependent which implies that angle between F and v is 90°. If v
changes (direction) the (directions)F should also change so that above condition is
satisfied.

(2 Marks Questions)

11. A long straight wire AB carries a current of 4A. A proton P travels at 4×10 6 M s-1parallel
to the wire 0.2 m from it a direction opposite to the current as shown in the current as
shown in the figure. Calculate the force which the magnetic field due to the current
carrying wire exert on the proton. Also specify its direction.

______________________________________________________________________________
______________________________________________________________________________

Er. Ujwal Kumar (Physics Mentor for NEET/ JEE-Mains, Adv/ KVPY/OLYMPIAD/CBSE)
______________________________________________________________________________
______________________________________________________________________________ 117
(3 Marks Questions)

12. Two particles A and B of masses m and 2m have charges q and 2q respectively. Both
these particle moving with velocity v1 and v2 respectively in the same direction enter the
same magnetic field B acting normally to their direction of motion. If the two forces F A
and FB acting on them are in the ration of 1:2, find the ratio of their velocities.
FA q1|(v ⃗⃗ 1 )|
⃗⃗ 1×B
Sol. =q
FB ⃗⃗ 2 )|
⃗⃗ 2×B
2|(v
1 qv1 Bsin90°
= (2q)v [∵ B1 = B2; magnetic field is same]
2 2 Bsin90°
v
1 = v1
2
∴ v1: v2 = 1: 1.

13. A proton, a deuteron and an alpha particle, are accelerated through the same potential
difference and then subjected to a uniform magnetic field B, perpendicular to the
direction of their motions. Compare
(i) their kinetic energies, and
(ii) if the radius of the circular path described by proton is 5 cm, determine the radii
of the path described by deuteron and alpha particles.
Sol. When a proton, a deuteron and an alpha particles are accelerated through potential
difference V, then their energies are
Ep = eV, Ed = eV, E = 2eV
(i) KEp = KEd: KE  = 1: 1: 2.
mv √2mKE
(ii) r = =
qB qB
√ mp √2mp √4mp
rp: rd: r = : : = 1: √2:1
e e 2e
As rp = 5cm, ∴ rd = 5√2 cm, r = 5 cm.

14. (a) Write the expression for the magnetic force acting on a charged particles moving with
velocity v in the presence of magnetic field B.
(b) A neutron, an electron and an alpha particle moving with equal velocities enter a
uniform magnetic field going into the plane of the paper as shown. Trace their paths in
the field and justify your answer?

Er. Ujwal Kumar (Physics Mentor for NEET/ JEE-Mains, Adv/ KVPY/OLYMPIAD/CBSE)
118

Sol. (a) Magnetic force acting on a charged particle q moving with a velocity v in a uniform
⃗)
magnetic field B is given by, 𝐹 = q(𝑣 × 𝐵
(b) Magnetic force on  particle. F = q𝑣 × 𝐵
⃗ =- 2e e B upward

So, curve will bend upward as force is perpendicular to the velocity.


Magnetic force on neutron, F = 0 (as q -= 0). So, neutron will move along straight line.
Magnetic force on electron, 𝐹e = q𝑣 × 𝐵⃗ = | - q v B| downwards
So, the curve will bend downwards as force is perpendicular to the velocity.
⃗ perpendicular to velocity
For a charged particle moving in a uniform magnetic field 𝐵
mv2 mv
qvB = ⇒r=
r qB
r is the radius of curved path. Here v = vn = ve = v
4me v 2me v
Radius of path traced by a-particle r = =
2eB eB
me v
Radius of path traced by electron re = eB

15. State the underlying principle of a cyclotron. Write briefly how this machine is used to
accelerate charged particle to high energies.
Sol. Cyclotron: It is a device by which positively charged particles like protons, deuterons,
etc. can be accelerated.

Er. Ujwal Kumar (Physics Mentor for NEET/ JEE-Mains, Adv/ KVPY/OLYMPIAD/CBSE)
Principle: A positively charged particle can be accelerated by making it to cross the same
electric field repeatedly with the help of a magnetic field. 119
Working and theory: At a certain instant, let D1 be positive and D2 be negative. The
radius of the circular path is given by qvB = mv2/r or r = mv/qB
2πr 2π mv 2πm
Period of revolution, T = = . =
v v qB qB
1 qB
Frequency of revolution, f = T = 2πm

16. (a) A point charge q moving with speed v enters a uniform magnetic field B that is
acting into the plane of paper as shown. What is the path followed by the Charge q and in
which plane does it moves?

(b) How does the path followed by the charge get affected if its velocity has a
component parallel?
(c) if an electric field E is also applied such that the particle continuous moving along
the original straight line path, what should be the magnitude and direction of the
electric field E.
Sol. (a) When a charged particle having charge q moves inside a magnetic field 𝐵 ⃗ with
velocity 𝑣 , it experiences a force.
𝐹 = q(𝑣 × 𝐵 ⃗)
When 𝑣 is perpendicular to 𝐵 ⃗ , the force 𝐹 on the charged particle provides the centripetal
force and makes it move along a circular path.
The point charge travels in the plane perpendicular to both 𝑣 and 𝐵 ⃗.
(b) If a component of velocity of the charge particle is parallel to the direction of the
magnetic field, then the force experienced due to that component will be zero, because F
= qvB sin 0° = 0 and particle will move in straight line. Also, the force experienced by
the component perpendicular to 𝐵 ⃗ moves the particle in a circular path. The combined
effect of both the components will move the particle in a helical path.
(c) The direction of the magnetic force is along negative y axis, so the direction of the
electric force should be along the positive Y axis to counter balance the magnetic force
and then the charge particle will move in the straight line path.
Therefore, the direction of electric field is along the positive Y axis and its magnitude is
given by E = vB.

17. In a chamber a uniform magnetic field of 6.5G (1G = 10-4T) is maintained. An electron is
shot into the field with a speed of 4.8×106 ms-1 normal to the field. Explain why the path

Er. Ujwal Kumar (Physics Mentor for NEET/ JEE-Mains, Adv/ KVPY/OLYMPIAD/CBSE)
of the electron is a circle. Determine the radius of the circular orbit. Given that e = 1.6 ×
10-19 C, me = 9.1 × 10-31kg. 120
Sol. The perpendicular magnetic field exerts a force on the electron perpendicular to its path.
This force continuously deflects the electron from its path and makes it moves along a
circular path.
∴ Magnetic force on the electron = Centripetal force
me v2 me v
evB sin 90° = or r =
r eB
Now B = 6.5G = 6.5 × 10-4 T, v = 4.8 × 106 ms-1
9.1×10−31 ×4.8×106
∴ r = 1.6×10−19×6.5×10−4 = 4.2×10-2m = 4.2cm

18. In previous question, obtain the frequency of revolution of the electron in its circular
orbit. Does the answer depend on the speed of the electron? Explain.
eB 1.6×10−19 ×6.5×10−4
Sol. Frequency of revolution of the electron in its circular orbit, f = 2πm = 2×3.14×9.1×10−31
6
= 18.18 × 10 HZ = 18 MHz.

19. An electron emitted by a heated cathode and accelerated through a potential difference of
2.0 kV, enters a region with uniform magnetic field of 0.15T. Determine the trajectory of
the electron if the field (i) is transverse to its initial velocity, (ii) makes and angle of 30°
with the initial velocity.
Sol. V = 2.0kV, = 2 × 103V, B = 0.15T, e = 1.6 × 10-19C., m = 9.1 × 10-31kg
Potential difference V imparts kinetic energy to the electron given by ½ mv2 = eV
2eV 2×1.6×10−19 ×2×103
Or Velocity gained by electron, v = √ =√ ms −1 = 2.65×107 ms-1.
m 9.1×10−31
mv 2
(i) When field ⃗B is transverse to the initial velocity v
⃗ , evB sin90° = r
mv 9.1×10−31 ×2.65×107
∴ r = eB = 1.6×10−19 ×0.15 m = 10-3 m = 1mm
Thus the electron follows a circular trajectory of radius 1mm around o the field B.
⃗ makes an angle 30° to the initial velocity v
(ii) When field B ⃗,
7 7 -1
v1 = vsin30° = 2.65 × 10 × ½ = 1.33 × 10 ms .
v2 = v cos 30°= 2.65 × 107 × 0.866 = 2.3 × 107 ms-1
mv1 mvsin30° 9.1×10−31 ×1.33×107
The radius of the helical path is given by r = = =
eB eB 1.6×10−19 ×0.15
= 50.4×10-5m = 0.50 mm

20. A magnetic field set up using Helmholtz coils is uniform in a small region and has a
magnitude of 075 T. In the same region, a uniform electrostatic field is maintained in a
direction normal to the common axis of the coils. A narrow beam of (single species)
charged particles all accelerated through 15kV enters this region in a direction
perpendicular to both the axis of the coils and the electrostatic field. If the beam remains
undeflected when the electrostatic field is 9.0 × 105 Vm-1, make a simple guess as to what
the beam contains. Why is the answer not unique?

Er. Ujwal Kumar (Physics Mentor for NEET/ JEE-Mains, Adv/ KVPY/OLYMPIAD/CBSE)
Sol. B = 0.75T, E = 9.0 × 105 Vm-1, V = 15kV = 15 × 103V
E 9.0×105 121
For undeflected beam, velocity of charged particles must be, e = = ms-1 =
B 0.75
12×105 ms-1.
But the kinetic energy of the charged particle is given by ½ mv2 = qV
q 1 v2 1 (12×105 )2
∴ m = 2 . V2 = 2 × Ckg-1 = 4.8 × 107 Ckg-1
15×103
q 1.6×10−19
Now for electrons, m = 2×1.67×10−27 = 4.8 × 107 Ckg-1.
Which means that the particle may be deuterons each of which contain one proton and
one neutron. The answer is not unique because we have determined only the ratio of
charge to mass. Other possible answer are He2+ and Li3+ etc.

(5 Marks Questions)

21. Particles of mass 1.6×10-27 kg and charge1.6×10-19 C are accelerated in a cyclotron of dee
radius 40cm. it employs a magnetic field 0.4 t. find the kinetic energy (in me v) of the
particle beam imparted by the accelerator.
q2 B2 R2 (1.6×10−19 )(0.4)2 (40×10−2)2
Sol. The kinetic energy imparted by accelerator, KE = =
2m 2×(1.6×10−27 )
-15
= 204.8 × 10 J = 1.28 MeV.

22. An  -particle is accelerated through a potential difference of 10 kV and moves along x-


axis. It enters in a region of uniform magnetic field B=2×10 -3T acting along y-axis. Find
the radius of its path.
Sol. Given V = 10 × 103V, B = 2 × 10-3T
2q∆V
Since charge is accelerated through V, Velocity = √ …(i)
m

mv2 mv m 2q∆V 2m∆V


Now when R enters a magnetic field, qvB = ;r = = qB √ =√
r qB m B2 q

For  particle, q = 2e, m = 6.4 × 10-27kg


2×6.4×10−27 ×104
∴ r = √(2×10−3 )2×2×1.6×10−19 = 10m

23. (a) Draw a schematic sketch of a cyclotron. Explain clearly the role of closed electric and
magnetic field in accelerating the charge. Hence derive the expression for the kinetic
energy acquired by the particles.
(b) An  -particles and a proton are released from the center of the cyclotron and made to
accelerate.
(i) Can both be accelerate at the same cyclotron frequency? Give reason to justify your
answer?

Er. Ujwal Kumar (Physics Mentor for NEET/ JEE-Mains, Adv/ KVPY/OLYMPIAD/CBSE)
(ii) When they are accelerate in turn, which of the two will have higher velocity at the
exit slit of the dees? 122

Sol. (a)

Electric field accelerates the particle when it passes through the gap and imparts energy
to charged particles. Magnetic field makes the charged particles to move in semicircular
paths.
Bqr
Velocity of particle, v = m
1 B2 q 2 r 2
∴ K = 2 mv 2 = 2m
(b) (i) No. The cyclotron frequency depends on the mass of the particle.
(ii) Proton.

D. FORCE ON CURRENT CARRYING CURRENT CONDUCTOR

(1 Mark Questions)

1. What is the magnitude of magnetic force per unit length on a wire carrying a current of
8A and making an angle of 30° with the direction of a uniform magnetic field of 0.15T?
Sol. Given I = 8A,  = 30°, B = 0.15T
F
As F = IlB sin , force per unit length, f = l = lBsinθ = 8 × 0.15 × sin 30° = 0.6 Nm-1.

2. Two long wires carrying current I1 and I2 are arranged as shown in Fig. The one carrying
current I1 is along is the x-axis. The other carrying current I2 is along a line parallel to the
y-axis given by x = 0 and z = d. Find the force exerted at O 2 because of the wire along the
x-axis.

Er. Ujwal Kumar (Physics Mentor for NEET/ JEE-Mains, Adv/ KVPY/OLYMPIAD/CBSE)
123

Sol. At O2 the magnetic field due to I1 is along the y axis. So the radius of path to the Dees’s
will remain unchanged.

(2 Marks Questions)

3. A 3.0 cm wire carrying a current of 10A is placed inside a solenoid perpendicular to its
axis. The magnetic field inside the solenoid is given to be 0.27T. What is the magnetic
force on the wire?
Sol. Given l = 3.0cm = 0.03m, I = 10A,  = 90°, B = 0.27T
F = IlB sin  = 10 × 0.03 = 0.27 × sin90° = 8.1 × 10-2N
The direction of the force is given by Fleming’s left hand rule.

4. Two long and parallel straight wires A and B carrying currents 8.0A and 5.0A in the
same direction are separated by a distance of 4.0cm. Estimate the force on a 10cm section
of wire A.
μ0 I 1 I 2 4π×10−7×8×5
Sol. Force per unit length of each wire is f = = = 2 × 10-5N.
2πr 2π×4×10−2

(3 Marks Questions)

5. Two infinitely long straight wires A1 and A2 carrying currents I and 2I following in the
same direction are kept ‘d’ distance apart. Where should a third straight wire A3 carrying
current 1.5 I be placed between A1 and A2 so that is experience no net force due to A1
and A2? Does the net force acting on A3 depend on the current flowing through it?
Sol.

Er. Ujwal Kumar (Physics Mentor for NEET/ JEE-Mains, Adv/ KVPY/OLYMPIAD/CBSE)
124

4π×10−7×I×1.5I
Force on A3 due to A1, f1 =
2πx
4π×10−7 ×2I×1.5I
Force on A3 due to A2, f2 = 2π(d−x)
When there is no net force on A3, f1 = f2
4π×10−7 ×I×1.5I 4π×10−7 ×2I×1.5I
= =
2πx 2π(d−x)
d – x = 2x ⇒ x = d/3
Hence from A1 at d/3 there is no net force on A3.

6. (a) Write an expression of magnetic moments associated with a current (I) carrying
circular coil of radius r having N turns.
(b)Concede the above mentioned coil placed in YZ plane with its center at the origin.
Derive expression for the value of fields due to it at point.
Sol. (a) The magnetic moment associated with a current (I) carrying a radius r having N
turns, is given by M = NIA = NIr2.
(b) Magnetic field at a distance x from the centre of the ring due to element dl, dB
μ0 idlsin90°
=
4π r2

Since angle between dl and r is 90°. The component dBcos will get cancelled due to
μ idl
symmetry, B = ∫ dbsinθ = ∫ (4π0 ) (sinθ)
r2
Here r and  are constants and sin = R/r
μ idl R μ idl
B = ∫ 4π0 ( r ) = ∫ 4π0
r2 r3
μ idl μ idl μ0 iR2
= 4π0 ∫ dl = 4π0 (2πR) =
r3 r3 2r2
μ iR2
Putting r = (R2 + x2)1/2, we get B = 2(R20+x2 )3/2

Er. Ujwal Kumar (Physics Mentor for NEET/ JEE-Mains, Adv/ KVPY/OLYMPIAD/CBSE)
μ NiR2
For N turns, B = 2(R20+x2 )3/2
125

7. A straight horizontal conducting rod of length 0.45 m and mass 60g is suspended by two
vertical wires at its ends. A current of 5.0A is set up in the rod through the wires.
(a) What magnetic field should be set up normal to the conductor in order that the tension
in the wires is zero?
(b) What will be the total tension in the wires if the direction of current is reversed,
keeping the magnetic field same as before? (Ignore the mass of the wires) g = 9.8 ms -2.
Sol. Here l = 0.45m, m = 60g = 0.06kg, I = 5.0A, g = 9.8 ms -2
(a) Tension in the supporting wires wil be zero when the weight of the rod is balanced by
the upward force IlB of the magnetic field, i.e. IlB = mg
mg 0.06×9.8
∴ B = Il = 5×0.45 T = 0.26T
So the horizontal magnetic field of 0.26T normal to the conductor must be applied in
such a direction that the Fleming’s left hand rule give s a magnetic force in the upward
direction.
(b) If the direction of current is reversed, the magnetic force will act in the downward
direction. Hence the total tension in the wires will be
T = 2 × the weight of the rod
= 2 × 0.06 × 9.8N = 1.176N

8. The wires which connect the battery of an automobile to its starting motor carry a current
of 300A (for a short time). What is the force per unit length between the wires if they are
70cm long and 1.5cm apart? Is the force attractive or repulsive?
Sol. I1 = I2 = 300A, r = 1.5cm = 1.5×10-2 m. l = 70cm = 0.70m.
μ I I
0 1 2 4π×10−7×300×300
The force per unit length between the wires is f = 2πr = 2π×1.5×10−2 Nm-1
= 1.2 Nm-1.
Total force between the wires, F = f × l = 1.2 × 0.70 = 0.84N
As the current in the two wires are in opposite directions, the force is repulsive.

9. A circular coil of 20 turns and radius 10cm is placed in a uniform magnetic field of 0.10T
normal to the plane of the coil. If the current in the coil is 5.0A, what is the
(a) total torque on the coil.
(b) total force on the coil
(c) average force on each electron in the coil due to the magnetic field?
(The coil is made of copper wire of cross-sectional area 10-5 m2 and the free electron
density in copper is given to be about (1029 m-3)).
Sol. N = 20, r = 10cm = 0.10m, B -= 0.10T, l = 50A,  = 0°
(a) Torque on the coil,  = NlBA sin  = 0 [since  = 0°]
(b) Magnetic forces on the opposite arms of coil are equal and opposite and at in the same
plane; hence the total force on the coil is zero.
BI
(c) Force on each electron is F = evB = nA [since I =enAv]
For given wire, n = 1029m-3, A = 10-5 m2
0.1×5
∴ F = 29 −5 N = 5×10-25N
10 ×10

Er. Ujwal Kumar (Physics Mentor for NEET/ JEE-Mains, Adv/ KVPY/OLYMPIAD/CBSE)
10. A solenoid 60cm long and of radius 4.0cm has 3 layers of windings of 300 turns each. A 126
2.0 cm long wire of mass 2.5g lies inside the solenoid near its centre normal to its axis;
both the wire and the axis of the solenoid are in the horizontal plane. The wire is
connected through two leads parallel to the axis of the solenoid to an external battery
which supplies a current of 6.0A in the wire. What value of current (with appropriate
sense of circulation) in the windings of the solenoid can support the weight of the wire?
G = 9.8 ms-2
Sol. Let I be the current in the windings of the solenoid which can support the weight of the
wire. The magnetic field inside the solenoid along its axis will be B = 0nl
Total number of turms 300×3
Here n = Length of the solenoid = 60×10−2 = 1500 turns m-1
∴ B = 4 ×10-7 × 1500 × I = 6 × 10-4 I tesla
This field acts as perpendicular to the current carrying wire. Therefore the magnetic force
on the wire will be F = l’lB = 6×(2×10-2)×6 × 10-4 I Newton.
The current I would support the wire if the above force equals the weight of the wire,
i.e. 6 × 2 × 10-2 × 6 ×10-4 l = 2.5 × 10-3 × 9.8
2.5×10−3×9.8
or I = 72×3.14×10−6 A = 108.3A

11. A long straight wire carrying current of 25A rests on a table as shown in Fig. Another
wire PQ of length 1m, mass 2.5 g carries the same current but in the opposite direction.
The wire PQ is free to slide up and down. To what height will PQ rise?

Sol. F = Bll sin  = Bll


μ0 I
B = 2πrh
μ0 I2 l
F = mg = 2πh
μ0 I 2 l 4π×10−7 ×250×25×1
h = 2πmg = 2π×2.5×10−3 ×9.8
= 51 × 10-4
h = 0.51 cm

(5 Marks Questions)

12. Two long straight parallel conductor carrying steady current I 1 and I2 are secreted by a
distance ‘d’. Explain briefly with the help of suitable diagram. How the magnetic field
due to one conductor acts on the other. Hence deduce the expression for the forcing
acting between the two conductors. Mention the nature of this force.

Er. Ujwal Kumar (Physics Mentor for NEET/ JEE-Mains, Adv/ KVPY/OLYMPIAD/CBSE)
Sol.
127

When two parallel infinite straight wires carrying currents I 1 and I2 are placed at a
distance d from each other, then current I 1 produces magnetic field, which at any point
μ0 I 1
on the second current carrying wire is B1 = directed inwards perpendicular to plane
2πd
of wires.
So, the current (I2) carrying wire then experiences a force due to this magnetic field
which on its length l is given by
⃗F21 = I2(l × ⃗B1)
μ0 I 1 μ0 I 1 I 2
F21 = F12 = I2lB1sin90° = I2l × or F21 = F123= l
2πd 2πd
The vector product (l × B⃗ 1) has a direction towards the wire carrying current I1. Hence
both the wires attract each other.
μ0 I 1 I 2
So, force per unit length that each wire exerts on the other is, f = .
2πd
μ
If I1 = I2 = 1A and d = 1m and l = 1m, then f = 2π0 = 2 × 10-7 m-1
Thus electric current through each of two parallel long wires placed at distance of 1m
from each other is said to be 1 ampere. If they exert a force of 2 × 10 -7 Nm-1 on each
other.

13. A uniform magnetic field of 1.5T exists in a cylindrical region of radius 10.0 cm, its
direction being parallel to the axis along east to west. A wire carrying current of 7.0A in
the north to south direction passes through this region. What is the magnitude and
direction of the force on the wire if
(i) the wire intersects the axis
(ii) the wire is turned from N-S to north east or north west direction
(iii) the wire in the N-S direction is lowered from the axis by a distance of 6.0 cm?
Sol. Here B = 1.5T, I = 7.0A.

Er. Ujwal Kumar (Physics Mentor for NEET/ JEE-Mains, Adv/ KVPY/OLYMPIAD/CBSE)
(i) As shown in figure, length of the wire in cylindrical region = diameter AB of
cylindrical region = 20cm = 0.20m 128
As the wires lies in NS direction and field acts along EW direction, so  = 90°
∴ Force on wire, F = IBl sin  = 7 × 1.5 × 0.20 × 1 = 2.1N
By Fleming’s left hand rule, this force acts in the vertically downwards direction.
(ii) When the wire turns from NS to NE or NW direction, suppose it makes angle q with
field B as shown in figure below. The length of wire in magnetic field, A’B’ = l’ say

l l
Clearly l′ = sinθ or l’ = sinθ
1
Force on wire, F = ll’sin = l. sinθ . Bsinθ = llB = 2.1N
This force acts in the vertically downward direction.
(iii) As shown in figure (i) when the wire is lowered by 6.0cm, length of the wire in the
magnetic field = 2x
But x = √102 − 62 = 8cm = 0.08m
∴ 2x = 0.16,  = 90°
Force on wire, F = llB = 7 × 0.16 × 0.15 = 1.68N
This force also acts in the vertically downward direction.

E. MAGNEITC DIPOLE MOMENT

(3 Marks Questions)

1. An electron of mass me revolves around a nucleus of charge +Ze . show that it behaves
e
like a tiny magnetic moments associated with it is expressed as µ   L where l is
2me
the orbital angular momentum of the electron. Give the signification of negative sign.
Sol.

Er. Ujwal Kumar (Physics Mentor for NEET/ JEE-Mains, Adv/ KVPY/OLYMPIAD/CBSE)
129

The electron of charge (- e) performs uniform circular motion around a stationary heavy
nucleus of charge +Ze. This constitutes a current I and forms a loop which behaves like a
tiny magnetic dipole.
I = e/T T = time period) …(i)
Let r be the orbital radius of the electron and v the orbital speed. Then,
T = 2r/v …(ii)
From (i) and (ii), I = ev/2r
Magnetic moment  = I r2 = evr/2
The direction of magnetic moment is into the plane of paper.
e e
Now,  = 2m (me vr) = 2m L where L is the magnitude of angular momentum of the
e e
electron.
−e
⃗ = 2m ⃗L
Vectorially, μ
e
The negative sign indicates the angular momentum of the electron is opposite in direction
to the magnetic moment.

2. (a) Show that the planner loop carrying a current I, having N closely wound turns and
area of cross-sectional A, possesses a magnetic moment m  NI A .
(b) When this loop is placed in a magnetic field B , find out the expression for the
torque acting on it.
Sol.

Er. Ujwal Kumar (Physics Mentor for NEET/ JEE-Mains, Adv/ KVPY/OLYMPIAD/CBSE)
Sol. When a rectangular loop PQRS if sides ‘a’ and ‘b’ carrying a current I is placed in
uniform magnetic field ⃗B, such that area vector ⃗A makes an angle  with direction of 130
magnetic field, then forces on the arms QR and Sp of loop are equal, opposite and
collinear, thereby perfectly cancel each other, whereas forces on the arm sPQ and RS of
loop are equal and opposite but not collinear, so they give rise to torque on the loop.
Force on side PQ or RS of loop is F = ibB sin 90° = lb B and perpendicular distance
between two non collinear forces is r1 = sin
So, torque τ = Fr1 -= ibB a sin  – I ba B sin
Or  = IAB sin  and if loop has N turns, then  = NIAB sin. In vector form, τ⃗ = ⃗M
⃗⃗ × ⃗B
Where M ⃗⃗⃗ = NIA
⃗ is called magnetic dipole moment of current loop and is directed in
direction area vector ⃗A i.e. normal to the plane of loop.

(a) If the plane of loop is normal to the direction of the magnetic field i.e.  = 0° between
⃗B and ⃗A then the loop does not experience any torque i.e., min = 0.
(b) If the plane of loop is parallel to the direction of magnetic field i.e.  = 90° between
⃗B and ⃗A then the loop experiences maximum torque, i.e., max = NIAB.

3. A current carrying circular loop of radius R is placed in the x-y plane with centre at the
origin. Half of the loop with x>0 is now bent so that it now lies in the y-z plane.
(a) The magnitude of the magnetic moment now diminishes
(b) The magnetic moment does not change.
(c) The magnitude of B at (0, 0, z), z>>R increases
(d) The magnitude of B at (0, 0, z), z>>R is unchanged.
Ans. (a)

F. TORQUE ON COIL AND GALVANOMETER

(1 Mark Questions)

Er. Ujwal Kumar (Physics Mentor for NEET/ JEE-Mains, Adv/ KVPY/OLYMPIAD/CBSE)
1. A circular current loop of magnetic moment M is in an arbitrary orientation in an
external magnetic field B. The work done to rotate the loop by 30° about an axis 131
perpendicular to its plane is
(a) MB (b) √3 MB/2 (c) Mb/2 (d) zero

Ans. (d)

2. How are figure of merit and current sensitivity of galvanometer related to each other?

______________________________________________________________________________
______________________________________________________________________________

3. If the current is increased by 1% in a moving coil galvanometer, what will be the


percentage increase in deflection?

______________________________________________________________________________
______________________________________________________________________________

(2 Marks Questions)

4. A square coil of the side 10cm consists of 20 turns and carries a current of 12A. The coil
is suspended vertically and normal to the plane of the coil and makes an angle of 30° with
the direction of a uniform horizontal magnetic field of magnitude 0.80T. What is the
magnitude of torque experienced by the coil?
Sol. Given A = 0.10m × 0.10m = 0.01m2, N = 20, I = 12A,  = 30°, B = 0.80T
Magnitude of torque is,  = NIBA sin  = 20 × 12 × 0.80 × 0.01 × sin 30° = 0.96Nm.

(3 Marks Questions)

5. (a) Define current sensitivity of a galvanometer. Write its expression.


(b) A galvanometer has resistance G and shows full scale deflection for current I g
(i)How can it be converted into an ammeter to measure current up to I 0
(ii) What is the effective resistance of this ammeter?
Sol. (a) Current sensitivity: It is defined as the deflection of coil per unit current flowing in it,
θ NAB
i.e., IS = I = k
(b) A galvanometer can be converted into an ammeter of given range by connecting a
suitable low resistance S called shunt in parallel to the given galvanometer, whose value
Ig
is given by S = (I ) G where Ig is the current for fill scale deflection of galvanometer,
0−Ig

I0 is the current to be measured by the galvanometer and G is the resistance of


galvanometer.

Er. Ujwal Kumar (Physics Mentor for NEET/ JEE-Mains, Adv/ KVPY/OLYMPIAD/CBSE)
132

In order to increase the rang of an ammeter n times, the value of shunt resistance to be
connected in parallel is S = G/(n – 1).

6. Two moving coils galvanometers M1 and M2 have the following particulars:


R1  10, N1  30, A1  3.6 103 m2 , B1  0.25T
R 2  14, N 2  42, A 2  1.8 103 m2 , B2  0.25T
The spring constants are identical for the two springs. Determine the ratio of (i) current
sensitivity and (ii) voltage sensitivity of M2 and M1.
Sol. Let torsion foe each meter= k
For a galvanometer, we have NIBA = k
α NBA
Its current sensitivity is defined as the deflection produced per unit current, i.e., =
I k
Current sensitivity of M2 N2 B2 A2 /k N2 B2 A2 42×0.50×1.8×10−3 7
∴ Current sensitivity of M = N =N = 30×0.25×3.6×10−3 = 5 = 1.4
1 1 B1 A1 /k 1 B1 A1
Voltage sensitivity of a galvanometer is define as the defelction produced per unit
α α NBA
voltage, i.e. V = IR = kR
Voltage sensitivity of M2 N2 B2 A2 /kR2 N B A R 7 10
=N = N2 B2 A2 × R1 = 5 × 14 = 1
Voltage sensitivity of M1 1 B1 A1 /kR1 1 1 1 2

7. (a) A circular coil of 30 turns and radius 8.0 cm carrying a current of 6.0A is suspended
vertically in a uniform horizontal magnetic field of magnitude 1.0T. The field lines make
an angle 60° with the normal to the coil. Calculate the magnitude of the counter torque
that must be applied to prevent the coil from turning.
(b) Would your answer change if the circular coil in (a) were replaced by a planar coil of
some irregular shape that encloses the same area?
Sol. (a) N = 20, r = 8.0cm = 0.08m, l = 6.0A, B = 1T,  = 60°
Magnitude fo counter torque = magnitude of defelcting torque
= NIBA sin = 30 × 6 × 1 × (3.14 × 0.08 × 0.08)sin 60°
= 30 × 6 × 3.14 × 64 × 10-4 × 0.866 = 3.1 Nm
(b) No, the answer would not change because the above formula fro the torque is true for
a planar loop of any slope.

8. A galvanometer coil has resistance of 12 and meter shows full scale deflection for a
current of 3 mA. How will you convert the meter into a voltmeter of range 0 to 18V?

Er. Ujwal Kumar (Physics Mentor for NEET/ JEE-Mains, Adv/ KVPY/OLYMPIAD/CBSE)
Sol. Here Rg= 12, Ig = 3mA = 3 × 10-3A, V = 18V
V 18
R = I − R g = 3×10−3 − 12 = 6000 – 12 = 5988 . 133
g
By connecting a resistance of 5988 in series with the given galvanometer we get a
voltmeter of range 0 to 18 V.

9. A galvanometer of resistance of 15 and the meter shows full scale deflection for a
current of 4mA. How will you convert the meter into an ammeter of range 0 to 6A?
Sol. Here Rg = 15, Ig = 4mA = 0.004A, I = 6A
I 0.004
RS= I−Ig ×Rg = 6−0.004 × 15 = 0.1010 = 10m.
g

By connecting a shunt of resistance 10m across the given galvanometer, we get an


ammeter of range 0 to 6A.

10. A galvanometer needs 50mV for a full scale deflection of 50 divisions. Find its voltage
sensitivity. What must be its resistance if its current sensitivity is 1 division/ A?
α 50 divisions 50 divisions
Sol. Voltage sensitivity, VS = V = 50 mV = 50×10−3 = 103div V-1
I 1 div μA−1 106 div A−1
Resistance of galvanometer, RS = VS = 103 div V−1 = 103 div V−1 = 1000.
S

(5 Marks Questions)

11. A square shaped current carrying loop MNOP is placed near a straight long current
carrying wire AB as shown in the figure the wire and the loop lie in the loop experience
a net force F toward the wire, find the magnitude of the force on the side NO of the loop.

Sol. The net force acting on the loop

⃗FNet = ⃗FPM – ⃗FNO = μ0 I1 I2 − μ0 I1 I2


2πL 4πL
μ0 I 1 I 2 μ I I
Now the force acting on the side NO, ⃗FNO = 4πL = 04π1 2 = FNet.

Er. Ujwal Kumar (Physics Mentor for NEET/ JEE-Mains, Adv/ KVPY/OLYMPIAD/CBSE)
12. A rectangular loop of wire of size 2.5 cm ×4cm carriers a steady current of 1 A. A
straight wire carrying 2A currents is kept near the loop as shown. If the loop and wire are 134
coplanar, find the (i) torque acting on the loop and (ii) the magnitude and direction of the
force on the loop due to the current carrying wire.

Sol. (i) Torque, τ⃗ = M ⃗ = MB sin . Here M and B are in the same direction, i.e.,  = 0°.
⃗⃗⃗ × B
∴ τ⃗ = 0.
(ii)

μ0 I 1 I 2 l
Force between two currents carrying wires, F = 2πr
μ0 ×2×1×4×10−2 2μ0
Force on arm AB, FAB = = N (Attractive towards the wire)
2π×2×10−2 π
μ0 ×2×1×4×10−2 8μ0
Force on arm CD, FCD = = N (Repulsive, away from the wire)
2π×4.5×10−2 9π
Forces on arms BC and DA are equal and opposite,. So, they cancel out each other.
μ0 8 10μ0 10×4π×10−7
Net force on the loop is F = FAB – FCD = [2 − ] = =
π 9 9π 9π
-7
= 4.44 × 10 N (Attractive towards the wire)

13. (a) Define the current sensitivity of a galvanometer.


(b) The coil area of a galvanometer is 16×10 -4 m2 it consist of 200 turns of a wire and
is in a magnetic field of 0.2T. the restoring torque constant of the suspension fiber
is 10-6 N m per degree. Assuming the magnetic field to be radial, calculate the
maximum current that can be measured by the galvanometer if the scale can
accommodate 30 deflections.
Sol. (a) Same as 5(a)
(b) A = 16 × 10-4 m2, N = 200, B = 0.2T, k = 10-6 Nm/degree,  = 30°,
k 10−6 ×30
I= θ= = 4.69×10-4A.
NBA 200×0.2×16×10−4

Er. Ujwal Kumar (Physics Mentor for NEET/ JEE-Mains, Adv/ KVPY/OLYMPIAD/CBSE)
14. (a) Explain using a labeled diagram, the principle and working of a moving coil
galvanometer. What is the function of (i) soft iron core? 135
(b) Define the terms (i) current sensitivity and (ii)voltage sensitivity of a galvanometer
why does increasing the current sensitivity not necessarily increase voltage sensitivity.
Sol. (a) A galvanometer is used to detect current in a circuit.

Principle and working: When current (I) is passed in the coil, torque  acts on the coil, is
given by  = NIAB sin  where  is the angle between the normal to plane of coil and the
magnetic field of strength B, N is the number of turns in a coil.
When the magnetic field is radial, as in case of cylindrical pole pieces and soft iron core,
then in every position of coil, the plane of the coil, is parallel to the magnetic field lines,
so that  = 90° and sin 90° = 1.
Deflecting torque,  = NIAB.
If C is torsional rigidity of the wire and  is the twist of suspension strip, then restoring
torque = C
NAB
Therefore  = I i.e.  ∝ I. Deflection of coil is directly proportional to current
C
flowing in the coil and hence we can construct a linear scale.
The uniform radial magnetic field keeps the plane of the coil always parallel to the
direction of the magnetic field, i.e. the angel between the plane of the coil and the
magnetic field is zero for all the orientations of the coil.
(b) (i) Same as 5 (a)
(ii) Voltage sensitivity: Current sensitivity/R. Thus on increasing the current sensitivity,
voltage sensitivity may or may not increase because of similar changes in the resistance
of the coil, which may also increase due to increase in temperature.

15. The scale of a galvanometer is divided into 150 equal divisions. The galvanometer has
the current sensitivity of 10 divisions per mA and the voltage sensitivity of 2 divisions
per mV. How the galvanometer can be designed to read (i) 6A division -1 and (ii) 1V per
division-1 ? [Ans. (i) 8.33×10-5  in parallel (ii) 9995 in series]

______________________________________________________________________________

Er. Ujwal Kumar (Physics Mentor for NEET/ JEE-Mains, Adv/ KVPY/OLYMPIAD/CBSE)
______________________________________________________________________________
______________________________________________________________________________ 136
______________________________________________________________________________
______________________________________________________________________________
______________________________________________________________________________
______________________________________________________________________________
______________________________________________________________________________
______________________________________________________________________________
______________________________________________________________________________
______________________________________________________________________________
______________________________________________________________________________

G. CASE STUDY

1. Motion of a Charged particle in a Uniform magnetic Field: The force experienced by


a particle on charge q moving with a velocity v in a uniform magnetic field B is given
by: F = q(v × B)
which is perpendicular to both v and B. Since F is perpendicular to v, no work is done on
the charged particle moving in a uniform magnetic field. If v is perpendicular to B, the
force cannot change the kinetic energy of the particle. Hence the magnitude of v cannot
change, only the direction of motion changes continuously. These are exactly the
conditions for uniform circular motion. Thus, if v is perpendicular to B, the charged
mv
particle follows a circular path whose radius is given by r = qB
qB
The frequency of revolution of the particle along the circular path is given by, v = 2πm
(i) A proton and an alpha particles are projected perpendicular to a uniform magnetic field
with equal velocities. The mass of an alpha particle is 4 times that of a proton and its
charge is twice that of a proton. If rp and r are radii of their circular path, then the ratio
rp/r is
1 1
(a) (b) (c) 1 (d) √2
√2 2
mv
Ans. (b) Use r = . The correct answer is (b).
qB
(ii) In Qs (i) what is the ratio rp/r if the two particles have equal kinetic energies before
entering the region of the magnetic field.
1
(a) 2 (b) 1 (c) 2 (d) 4
2K mv 1
Ans. (b)b Kinetic energy K = ½ mv2 which gives v = √ m . Hence r = r = = qB √2mK
qB
Using this relation we find the t the correct choice is (b).
(iii) In Qs (i) what is the ratio rp/r if the two particles have equal linear momenta before
entering the region of the magnetic field.
(a) 1 (b) √2 (c) 2 (d) 2√2

Er. Ujwal Kumar (Physics Mentor for NEET/ JEE-Mains, Adv/ KVPY/OLYMPIAD/CBSE)
mv p
Ans. (c) Linear momentum, p = mv. Hence r = r = = qB. Using this relation we find that the
qB
137
correct choice is (c).
(iv) In Qs (i) what is the ratio rp/r if the two particles are accelerated through the same
potential difference before entering the region of the magnetic field
1
(a) √2 (b) 1 (c) √2 (d) 2
Ans. (a) Kinetic energy gained by a particle of charge q when it is accelerated through a
potential difference V is K = qV
1 1 2mV
Hence qB √2mK = B √ . Using this relation we find that the correct choice is (a).
1
(v) Which of the following particles in motion cannot be deflected by magnetic fields?
(a) Protons (b) Beta particles (c) Alpha particles (d) Neutrons
Ans. (d). The correct choice is (d) because neutrons have no charge.

2. The cyclotron: The cyclotron which is used to accelerate charged particles such as
protons, deuterons, alpha particles, etc, to a very high energies. The principle on which a
cyclotron works is based on the fat that an electric field can accelerate a charged particle
and a magnetic field can throw it into a circular orbit. A particle of charge +q
experiences a force qE in an electric field E and this force is independent of the velocity
of the particle. The particle is accelerated in the direction of the field. On the other hand,
a magnetic field at right angles to the direction of motion of the particle throws the
particle in a circular orbit in which the particle revolves with a frequency that doe no
depend on its speed. A modest potential difference is used as a source of electric field. If
a charged particle is made to pass through this potential difference a number of times, it
will acquire an enormously large velocity and hence kinetic energy.
(i) Which of the following cannot be accelerated in a cyclotron?
(a) Protons (b) Deuterons (c) Alpha particles (d) Neutrons
Ans. (d) The correct choice is (d) because neutrons have no charge.
(ii) The working of cyclotron is based on the fact that
(a) the force experienced by a charged particle in an electric field is independent of its
velocity.
(b) the radius of the circular orbit of a charged particle in a magnetic field increases with
increase in its speed.
(c) at a given speed, the radius of the circular orbit is the same for particles having the
same charge to mass ratio.
(d) the frequency of revolution of the particle along the circular path does not depend on
its speed
Ans. (d)
(iii) Cyclotron is not suitable for accelerating
(a) electrons (b) protons (c) deuterons (d)alpha paritcles

Er. Ujwal Kumar (Physics Mentor for NEET/ JEE-Mains, Adv/ KVPY/OLYMPIAD/CBSE)
Ans. (a). The correct choice is (a). Because of extremely small mass, the frequency of
circulation of electrons is very high. Oscillators of such high frequencies are not easily 138
obtainable.

H. ASSERTION REASON TYPE QUESTIONS


(a) If both assertion and reason are true and reason is the correct explanation of assertion.
(b) If both assertion and reason are true but reason is not the correct explanation of
assertion.
(c) If assertion is true but reason is false (d) If both assertion and reason are false
(e) If assertion is false but reason is true.
1. Assertion: If an electron is not deflected while passing through a certain region of space,
then only possibility is that there is no magnetic field in this region.
Reason: Force is directly proportional to magnetic field applied.
Ans. (e) Assertion is false but reason is true
In this case we cannot be sure about the absence of the magnetic field because if the
electron moving parallel to the direction of magnetic field, the angle between velocity
and applied magnetic field is zero and the force experienced by the electron is zero (F =
0). Then also electron passes without deflection.
2. Assertion: Free electrons always keep on moving in a conductor even then no magnetic
force act on them in magnetic field unless a current is passed through it.
Reason: The average velocity of free electron is zero.
Ans. (a) Both assertion and reason are true and reason is the correct explanation of assertion.
In the absence of the electric current, the free electrons in a conductor are in a state of
random motion, like molecule in a gas. Their average velocity is zero i.e. they do not
have any net velocity in a direction. As a result, there is no net magnetic force on the free
electrons in the magnetic field. On passing the current, the free electrons acquire drift
velocity in a definite direction, hence magnetic force acts on them, unless the field has no
perpendicular component.
3. Assertion: Out of galvanometer, ammeter and voltmeter, resistance of ammeter is lowest
and resistance of voltmeter is highest.
Reason: An ammeter is connected in series and a voltmeter is connected in parallel in a
circuit.
Ans. (b) Both assertion and reason are true but reason is not the correct explanation of
assertion.
In a voltmeter, a high resistance is connected in series with a galvanometer. That is why
resistance of voltmeter is highest. In an ammeter, a low resistance in parallel with a
galvanometer. That is why resistance of ammeter is lowest.
4. Assertion: For a point on the axis if a circular coil carrying current, magnetic field is
maximum at the centre of the coil.
Reason: magnetic field is inversely proportional to the distance of point from the circular
coil.
Ans. (a) Both assertion and reason are true and reason is the correct explanation of assertion.
The magnitude of the magnetic field produced due to flow of current through a circular
μ0 2πIa2
loop at a point on its axis is given by B =
4π (a2 +x2 )3/2

Er. Ujwal Kumar (Physics Mentor for NEET/ JEE-Mains, Adv/ KVPY/OLYMPIAD/CBSE)
This is maximum at centre of loop, where x = 0
μ 2πI
B = 4π0 =
μ0 I 139
a 2a
5. Assertion: A solenoid tends to expand, when a current passes through it.
Reason: Two straight parallel metallic wires carrying current in same direction repel each
other.
Ans. (d) Both assertion and reason are false.
When current flows through a solenoid, the currents in the various turns of the solenoid
are parallel and in the same direction,. Since the currents flowing through parallel wires
in the same direction lead to force of attraction between them, the turns of the solenoid
will also attract each other and as a result, the solenoid tends to contract.

I. CHALLENGING PROBLEMS

1. A magnetic field of 100 G (1G = 10 -4 T) is required which is uniform in a region of linear


dimension about 10cm and area of cross-section about 10-3 m2. The maximum current
carrying capacity of a given coil of wire is 15A and the number of turns per unit length
that can be wound round a core is at most 1000 turns m-1, Suggest some appropriate
design particulars of a solenoid for the required purpose. Assume the core is not
ferromagnetic.
Sol. Here B = 100G = 10-2T, l = 15A, n = 1000 turns m-1
Magnetic field inside a solenoid, B = 0nI
B 10−2
∴ nI = μ = 4π×10−7 = 7955 ≈ 8000
0
We may take l = 10A, then n – 800
The solenoid may have length 50cm and area of square section 5 × 10 -2 m2 (five times the
given value) so as to avoid edge effects, etc.

2. For a circular coil of radius R and N turns carrying current I, the magnitude of the
magnetic field at a point on its axis at a distance x from its centre is given by
0 IR 2 N
B
2(x 2  R 2 )3/ 2
(a) Show that this reduces to the familiar result for field at the centre of the coil.
(b) Consider two parallel co-axial circular coils of equal radius R, and number of turns N,
carrying equal currents in the same direction, and separated by a distance R. Show that
the field on the axis around the mid-point between the coils is uniform over a distance
that is small as compared to R and is given by
 NI
B  0.72 0
R
Such an arrangement used to produce a nearly uniform magnetic field over a small
region is known as Helmholtz coils.

Er. Ujwal Kumar (Physics Mentor for NEET/ JEE-Mains, Adv/ KVPY/OLYMPIAD/CBSE)
μ IR2 N
Sol. (a) Given B = 2(x20+R2 )3/2
140
μ IR2 N μ IN
At the centre of the coil, x = 0, so B = 02R3 = 2R 0

This is the standard result for the field at the centre of the coil.
(b) As shown in figure, consider a small region of length 2d about the midpoint O
between the two coils.

μ0 IR2 N
Given B =
2(x2 +R2 )3/2
μ0 IR2 N
Therefore, the magnetic field at the point P due to coil 1, B 1 = 2 3/2
, acting
R
2[R2 +( +d) ]
2
along PO2.
μ0 IR2 N
Magnetic field at point P due to coil 2, B2 = 2 3/2
acting along PO2
R
2[R2 +( −d) ]
2
Total magnetic field at the point P will be B = B1 + B2
μ0 IR2 N 1 1
= [ 3/2 + 3/2 ]
2 R2 R2
(R2 + +d2+Rd) (R2 + +d2 −Rd)
4 4

μ0 IR2 N 1 1
= [ 3/2 + 3/2 ] [Neglecting d2, as d <<R]
2 5R2 5R2
( +Rd) ( −Rd)
4 4

μ0IR2 N 1 1
= 3/2 [ 4d 3/2
+ 4d 3/2
]
5R2
2( ) (1+ ) (1− )
4 5R 5R

μ0 IN 4 3/2 4d −3/2 4d −3/2


= ( ) [(1 + ) + (1 − ) ]
2R 5 5R 5R
μ IN 4 3/2
0 6d 6d
= 2R (5) [(1 − 5R) + (1 + 5R)] [Expanding by binomial theorem and neglecting
higher powers of d/R]
μ0 IN
Or B = 0.72 2R
Magnetic field will also be same at the point Q. In fact, it wil be uniform over the small
region of length 2d around the midpoint O.

3. A galvanometer with a coil of resistance 12.0 shows full scale deflection for a current
of 25 mA. How will you convert the meter into: (i) an ammeter of range 0 to 7.5A (ii) a
voltmeter of range 0 to 10.0V.
Determine the net resistance of the meter in each case. When an ammeter is put in a
circuit, does it read (slightly) less or more than the actual current in the original circuit?

Er. Ujwal Kumar (Physics Mentor for NEET/ JEE-Mains, Adv/ KVPY/OLYMPIAD/CBSE)
When a voltmeter is put across a part of the circuit, does it read (slightly) less than or
more than the original voltage drop? Explain. 141
Sol. (i) For conversion into ammeter: Rg = 12, Ig = 2.5mA = 0.0025A, I = 7.5A
Ig 0.0025 2.5×12×10−3
RS = I−I × R g = 7.5−0.0025 × 12 = = 4.0×10-3
g 7.4975

So by connecting a shunt resistance of 4.0 × 10-3 in parallel with the galvanometer, we


get an ammeter of range 0 to 7.5A.
1 1 1 3001
Net resistance RA is given by: R = 12 + 4×10−3 =
A 12
12
Or RA = 3001  = 4×10 . -3

When an ammeter is put in a circuit, it reads slightly less than the actual current in the
original circuit because a very small resistance is introduced in the circuit.
(ii) For conversion into voltmeter: Rg = 12, Ig = 2.5 × 10-3A. V = 10V
V 10
∴R= − Rg = − 12
Ig 2.5×10−3

= 4000 – 12 = 3988.
So, by connecting a resistance of 3988 in series with the galvanometer, we get a
voltmeter of range 0 to 10V.
Now resistance, RV = (3988+ 12) = 4000.
Because voltmeter draws small current for its deflection, so it reads slightly less the
original voltage drop.

SPACE FOR ROUGH WORK

Er. Ujwal Kumar (Physics Mentor for NEET/ JEE-Mains, Adv/ KVPY/OLYMPIAD/CBSE)
142

SPACE FOR NOTES

Er. Ujwal Kumar (Physics Mentor for NEET/ JEE-Mains, Adv/ KVPY/OLYMPIAD/CBSE)

You might also like